Further evidence bearing on Jamison's activities must have come to light. On the basis of previously available eviden...

tselimovic on September 16, 2014

Explanation

Why is B the correct answer? Is there a particular way to set this question up?

Reply
Create a free account to read and take part in forum discussions.

Already have an account? log in

Naz on September 23, 2014

The conclusion in the argument is: "Further evidence bearing on Jamison's activities must have come to light."

Why? We know that based solely on the previously available evidence, it would have been impossible to prove that Jamison was a party to the fraud. We also know that Jamison's active involvement in the fraud has now been definitively established.

We can rewrite this: if it was proven that Jamison was a party to the fraud, then more than just the previously available evidence must have come to light.

PR: PF ==> ME
not ME ==> not PF

We know that Jamison's active involvement in the fraud has now been definitively established.

P: PF

Thus, more evidence must have come to light.

C: ME

The stimulus presents us with a principle rule and then invokes the sufficient condition of that principle rule to conclude the necessary condition. This is a valid Sufficient & Necessary argument.

Answer choice (B) states: "Turner must not have taken her usual train to Nantes today. Had she done so, she could not have been in Nantes until this afternoon, but she was seen having coffee in Nantes at 11 o'clock this morning."

Let's diagram this:

If Turner had taken her usual train to Nantes, then she could not have been in Nantes until this afternoon.

PR: TUT ==> not NBA (NBA = in Nantes before afternoon)
NBA ==> not TUT

"she was seen having coffee in Nantes at 11 o'clock this morning."

P: NBA

"Turner must not have taken her usual rain to Nantes today."

C: not TUT

Just as in the stimulus, we are given a principle rule and then the sufficient condition of that rule is invoked to conclude the necessary condition. So, as in the stimulus, we have a valid Sufficient & Necessary argument. Therefore, answer choice (B) is the correct answer.

Hope that was helpful! Please let us know if you have any other questions.